Discussioni su calcolo di variabile complessa, distribuzioni, Trasformata di Fourier, Teoria della misura, Analisi funzionale, Equazioni alle derivate parziali, Calcolo delle Variazioni e oltre.

Regole del forum

Consulta il nostro regolamento e la guida per scrivere le formule
Rispondi al messaggio

Funzione armonica limitata e non costante

03/01/2023, 13:57

Trovare una funzione \( f: \mathbb{Z}^3 \to \mathbb{R} \) che sia discreta armonica su \( \mathbb{Z}^3 \) tranne che in \( (0,0,0) \) e che sia limitata ma non costante.

Non riesco proprio. Una funzione armonica ha la proprietà che il massimo/minimo sta ai bordi quindi ho pensato che ad un certo punto deve essere periodoca in modo strano e oscillare e il massimo dev'essere nel orogine

Re: Funzione armonica limitata e non costante

11/01/2023, 15:29

E buh. Ci devi dire almeno che cosa significa "discreta armonica". Il Laplaciano discreto com'è definito? Non è che tutti possiamo avere queste robe specialistiche fresche in testa come te. Lo stesso vale per gli altri tuoi post, ormai sei arrivato ad un livello di specializzazione molto alto.

Comunque, per dire una cavolata tanto per iniziare la discussione, la prima cosa che proverei a fare è considerare
\[
f(x, y, z)=\begin{cases} 1, &(x, y, z)=(0,0,0), \\ 0, & (x, y, z)\ne (0,0,0). \end{cases}\]
Immagino che questa non sia discreta armonica su \(\mathbb Z^3\setminus\{(0,0,0)\}\), altrimenti il problema sarebbe risolto. Ma perché? Si può modificare l'esempio un pochino?

Re: Funzione armonica limitata e non costante

11/01/2023, 19:07

Dato un grafo \(G = (V,E) \) diciamo che una funzione \( f: V \to \mathbb{R} \) è discreta armonica se vale che
\[ \Delta f(y) = \frac{1}{\deg y} \sum_{x \sim y} ( f(x)-f(y) ) = 0 \]
O in altre parole
\[ f(y) = \frac{1}{\deg y} \sum_{x \sim y} f(x) \]
Dove \( \Delta \) è il Laplaciano discreto, \( x \sim y \) significa che \(x \) è connesso con un arco a \(y \).

Nel nostro caso abbiamo
\[ f(x,y,z) = \frac{1}{6} \left( f(x+1,y,z) + f(x-1,y,z) + f(x,y+1,z) + f(x,y-1,z) + f(x,y,z+1)+f(x,y,z-1) \right) \]

La funzione che hai detto te non è armonica perché ad esempio \( (1,0,0) \) possiede 6 vicini tra cui \((0,0,0) \) ora hai che
\[ \Delta f(1,0,0) = - \frac{1}{6} \]

Idea:
Credo che la soluzione ad una PDE discretizzata simile alla seguente verifica il mio problema

\[ \left\{\begin{matrix}
\Delta f(x) = 0 & x \in \mathbb{Z}^3 \setminus \{ (0,0,0) \} \\
f(x)= \delta_{x=y} & y \in \{(0,0,0)\}
\end{matrix}\right. \]
La soluzione a questo problema è unica, perché se ne esistono due, diciamo \(f,g\), allora \( h=f-g \) è soluzione di
\[ \left\{\begin{matrix}
\Delta h(x) = 0 & x \in \mathbb{Z}^3 \setminus \{ (0,0,0) \} \\
h(x)=0 & x \in \{ (0,0,0)\}
\end{matrix}\right. \]
ma per il principio del massimo/minimo per una funzione armonica abbiamo che il massimo di \(h(x) \) sta sul bordo, i.e. in \((0,0,0)\), pertanto \( h=0 \) ovunque e \(f=g \).

La soluzione del problema originale è data dalla misura armonica discreta, i.e. \( f(x)= H(x,y) \), ora la misura armonica discreta descrive la soluzione a questo problema
Sia \( X= (X_n)_{n \geq 0} \) una passeggiata aleatoria semplice in \( \mathbb{Z}^3 \) che comincia in \(x \in \mathbb{Z}^3 \) e sia \( \tau = \inf\{ n \geq 0 : X_n = y \} \) dove in questo caso poniamo \( y= (0,0,0) \). \(H(x,y)\) ci dà la probabilità che incominciando da \(x\) usciamo per la prima volta da \(y\). Abbiamo quindi che
\[ H(x,y) = \mathbb{P}_x [ X_{\tau} = y ] \]
infatti se \(x = y \) allora chiaramente la probabilità è \(1\) (perché incomincia già dal origine), inoltre siccome è una probabilità abbiamo che è bounded, e chiaramente è armonica, resta da dimostrare che non è costante e credo che lo sia perché una passeggiata aleatoria su \( \mathbb{Z}^3 \) che inizia in \(x\) non torna in \(x\) con probabilità \( > 0 \), quindi c'è una probabilità \( >0 \) che la passeggiata aleatoria non visiti mai \(y\) siccome se visitasse \( y \) con probabilità \(1 \) allora avremmo che siccome è una Markov Chain, per la Markov Chain property (perdità di memoria) abbiamo che con probabilità \(1 \) torna ad \(x\).

Re: Funzione armonica limitata e non costante

11/01/2023, 21:02

Ma la soluzione fondamentale non riesci a calcolarla direttamente? Devi davvero fare tutto quel discorso probabilistico?

Re: Funzione armonica limitata e non costante

11/01/2023, 23:39

Mah... io non riesco

Re: Funzione armonica limitata e non costante

14/02/2023, 17:34

Ho visto che hai fatto un cross-post su Math.SE:

https://math.stackexchange.com/q/4610766/8157

Non c'è problema, anzi hai fatto bene. Ma è una cosa minima di educazione postare il link al cross-post. Fallo sempre le prossime volte per favore.

Re: Funzione armonica limitata e non costante

14/02/2023, 17:48

Volevo poi aggiungere un commento. Questo problema è molto interessante perché descrive un fenomeno discreto che non ha un analogo continuo. Infatti una funzione armonica su \(\mathbb R^d\setminus\{0\}\) limitata è automaticamente costante.

Dimostrazione: Sia \(u\in C^\infty(\mathbb R^d\setminus\{0\})\) armonica, ovvero, \(\Delta u(x)=0\) per ogni \( x\ne 0\). Se \(u\) è limitata allora \(u\) ha una estensione armonica a \(\mathbb R^d\): S. Axler, Harmonic Function Theory, Theorem 2.3. (La dimostrazione usa l'integrale di Poisson. In sostanza, detta \(B\) la palla aperta di raggio 1, si ha che \(P[u|_{\partial B}] = u\) su \(B\)).

Assodato questo la dimostrazione è finita: \(u\) è una funzione armonica limitata su tutto \(\mathbb R^d\) e quindi è costante per il teorema di Liouville. \(\Box\)

Re: Funzione armonica limitata e non costante

16/02/2023, 15:56

dissonance ha scritto:Ho visto che hai fatto un cross-post su Math.SE:

https://math.stackexchange.com/q/4610766/8157

Non c'è problema, anzi hai fatto bene. Ma è una cosa minima di educazione postare il link al cross-post. Fallo sempre le prossime volte per favore.

Mi scuso se è stato interpretato come non educazione! Va bene! :wink:

dissonance ha scritto:Volevo poi aggiungere un commento. Questo problema è molto interessante perché descrive un fenomeno discreto che non ha un analogo continuo. Infatti una funzione armonica su \( \mathbb R^d\setminus\{0\} \) limitata è automaticamente costante.

Dimostrazione: Sia \( u\in C^\infty(\mathbb R^d\setminus\{0\}) \) armonica, ovvero, \( \Delta u(x)=0 \) per ogni \( x\ne 0 \). Se \( u \) è limitata allora \( u \) ha una estensione armonica a \( \mathbb R^d \): S. Axler, Harmonic Function Theory, Theorem 2.3. (La dimostrazione usa l'integrale di Poisson. In sostanza, detta \( B \) la palla aperta di raggio 1, si ha che \( P[u|_{\partial B}] = u \) su \( B \)).

Assodato questo la dimostrazione è finita: \( u \) è una funzione armonica limitata su tutto \( \mathbb R^d \) e quindi è costante per il teorema di Liouville. \( \Box \)

Sì, inoltre nel caso di una funzione armonica discreta è vero solo per \( \mathbb{Z}^d \) con \(d \geq 3 \), infatti per \(d \leq 2 \) non è possibile perché le passeggiate aleatorie semplici sono ricorrenti.

https://math.stackexchange.com/questions/4610889/bounded-functions-f-mathbb-zn-to-mathbb-r-that-are-discrete-harmonic-at-all

Re: Funzione armonica limitata e non costante

16/02/2023, 16:22

Inoltre vorrei aggiungere un ulteriore commento, il fatto che le funzioni "armoniche" e limitate su \( \mathbb{Z}^d \) sono costanti è strettamente legato al fatto che \( \mathbb{Z}^d\) è un gruppo amenabile. Infatti se esiste una probabilità \( \mu\) che rende lo spazio delle funzioni limitate e \(\mu\)-armoniche su un gruppo \(G\) costanti allora \(G\) è amenabile, inoltre è un equivalenza se assumi che il gruppo è numerabile. In generale puoi dimostrare questa cosa

Sia \(G\) un gruppo e \( \mu \in \operatorname{Prob}(G) \) una probabilità su \(G\), i.e. \( \operatorname{Prob}(G) = \{ \mu \mid \mu: G \to [0,1] \text{ tale che } \sum_{g \in G} \mu (g) =1 \} \).
Sia \( f \in \ell^{\infty}(G) \), diciamo che \(f\) è \( \mu\)-armonica se \( f \ast \mu = f \), dove
\[ f \ast \mu (x) = \sum_{g \in G} f(xg^{-1})\mu(g) \]

1) Supponiamo che \( f \) raggiunge il suo massimo in \(g_m \in G \) e supponiamo che per ogni \( g \in G \) esistono \( g_1,\ldots, g_n \in \operatorname{supp}(\mu) \) tale che \(g= g_1 \cdot \ldots \cdot g_n \). Si può dimostrare che se \(f\) è \(\mu\)-armonica allora è costante.

2) Si può trovare un controesempio tale che si ha solo che \(f\) è limitata.

3) Si può trovare un controesempio con \(f\) che raggiunge il suo massimo in un certo \(g_m \in G\) e tale che si ha solo \( \left< \operatorname{supp}(\mu) \right> = G \).

NB: nel caso di \( \mathbb{Z} \) possiamo prendere \( \mu = \frac{1}{2} \left( \delta_{-1} + \delta_{1} \right) \) e la definizione coincide con le funzioni armoniche discrete su \( \mathbb{Z}\). Attenzione però non è vero per tutte le probabilità.
Rispondi al messaggio


Skuola.net News è una testata giornalistica iscritta al Registro degli Operatori della Comunicazione.
Registrazione: n° 20792 del 23/12/2010.
©2000— Skuola Network s.r.l. Tutti i diritti riservati. — P.I. 10404470014.